Difference between revisions of "2003 AMC 10A Problems/Problem 12"

m (Problem)
m (Solution)
(4 intermediate revisions by 4 users not shown)
Line 1: Line 1:
 
== Problem ==
 
== Problem ==
A point <math>(x,y)</math> is randomly picked from inside the rectangle with  vertices <math>(0,0)</math>, <math>(4,0)</math>, <math>(4,1)</math>, and <math>(0,1)</math>. What is the probability that <math>x>y</math>?  
+
A point <math>(x,y)</math> is randomly picked from inside the rectangle with  vertices <math>(0,0)</math>, <math>(4,0)</math>, <math>(4,1)</math>, and <math>(0,1)</math>. What is the probability that <math>x<y</math>?  
  
 
<math> \mathrm{(A) \ } \frac{1}{8}\qquad \mathrm{(B) \ } \frac{1}{4}\qquad \mathrm{(C) \ } \frac{3}{8}\qquad \mathrm{(D) \ } \frac{1}{2}\qquad \mathrm{(E) \ } \frac{3}{4} </math>
 
<math> \mathrm{(A) \ } \frac{1}{8}\qquad \mathrm{(B) \ } \frac{1}{4}\qquad \mathrm{(C) \ } \frac{3}{8}\qquad \mathrm{(D) \ } \frac{1}{2}\qquad \mathrm{(E) \ } \frac{3}{4} </math>
Line 11: Line 11:
 
The line <math>x=y</math> intersects the rectangle at <math>(0,0)</math> and <math>(1,1)</math>.  
 
The line <math>x=y</math> intersects the rectangle at <math>(0,0)</math> and <math>(1,1)</math>.  
  
The area which <math>x>y</math> is the right isosceles triangle with side length <math>1</math> that has vertices at <math>(0,0)</math>, <math>(1,1)</math>, and <math>(0,1)</math>.   
+
The area which <math>x<y</math> is the right isosceles triangle with side length <math>1</math> that has vertices at <math>(0,0)</math>, <math>(1,1)</math>, and <math>(0,1)</math>.   
  
 
The area of this triangle is <math>\frac{1}{2}\cdot1^{2}=\frac{1}{2}</math>  
 
The area of this triangle is <math>\frac{1}{2}\cdot1^{2}=\frac{1}{2}</math>  
  
Therefore, the probability that <math>x<y</math> is <math>\frac{\frac{1}{2}}{4}=\frac{1}{8} \Rightarrow A</math>
+
Therefore, the probability that <math>x<y</math> is <math>\frac{\frac{1}{2}}{4}=\frac{1}{8} \Rightarrow \boxed{\mathrm{(A)}\ \frac{1}{8}}</math>
  
 
== See Also ==
 
== See Also ==
Line 22: Line 22:
 
[[Category:Introductory Algebra Problems]]  
 
[[Category:Introductory Algebra Problems]]  
 
[[Category:Introductory Geometry Problems]]
 
[[Category:Introductory Geometry Problems]]
 +
{{MAA Notice}}

Revision as of 13:14, 28 November 2015

Problem

A point $(x,y)$ is randomly picked from inside the rectangle with vertices $(0,0)$, $(4,0)$, $(4,1)$, and $(0,1)$. What is the probability that $x<y$?

$\mathrm{(A) \ } \frac{1}{8}\qquad \mathrm{(B) \ } \frac{1}{4}\qquad \mathrm{(C) \ } \frac{3}{8}\qquad \mathrm{(D) \ } \frac{1}{2}\qquad \mathrm{(E) \ } \frac{3}{4}$

Solution

The rectangle has a width of $4$ and a height of $1$.

The area of this rectangle is $4\cdot1=4$.

The line $x=y$ intersects the rectangle at $(0,0)$ and $(1,1)$.

The area which $x<y$ is the right isosceles triangle with side length $1$ that has vertices at $(0,0)$, $(1,1)$, and $(0,1)$.

The area of this triangle is $\frac{1}{2}\cdot1^{2}=\frac{1}{2}$

Therefore, the probability that $x<y$ is $\frac{\frac{1}{2}}{4}=\frac{1}{8} \Rightarrow \boxed{\mathrm{(A)}\ \frac{1}{8}}$

See Also

2003 AMC 10A (ProblemsAnswer KeyResources)
Preceded by
Problem 11
Followed by
Problem 13
1 2 3 4 5 6 7 8 9 10 11 12 13 14 15 16 17 18 19 20 21 22 23 24 25
All AMC 10 Problems and Solutions

The problems on this page are copyrighted by the Mathematical Association of America's American Mathematics Competitions. AMC logo.png